From a7323c0c4ccaeb9e432130c485833aacefbb52bd Mon Sep 17 00:00:00 2001 From: "W. Trevor King" Date: Wed, 11 Apr 2012 13:28:23 -0400 Subject: [PATCH] Fix problem* vs. problem62 in Serway and Jewett v4's problem 1.62. --- latex/problems/Serway_and_Jewett_4/problem01.62.tex | 2 +- 1 file changed, 1 insertion(+), 1 deletion(-) diff --git a/latex/problems/Serway_and_Jewett_4/problem01.62.tex b/latex/problems/Serway_and_Jewett_4/problem01.62.tex index 447af74..c81831c 100644 --- a/latex/problems/Serway_and_Jewett_4/problem01.62.tex +++ b/latex/problems/Serway_and_Jewett_4/problem01.62.tex @@ -7,7 +7,7 @@ Demonstrate that for small angles ($< 20\dg$) where $\alpha$ is in radians and $\alpha '$ is in degrees. Use a calculator to find the largest angle for which $\tan \alpha$ may be approximated by $\alpha$ with an error less than $10.0$\%. -\end{problem62} % Problem 1.62 +\end{problem*} \begin{solution} To kill both birds with one stone, -- 2.26.2